LSAT and Law School Admissions Forum

Get expert LSAT preparation and law school admissions advice from PowerScore Test Preparation.

 Adam Tyson
PowerScore Staff
  • PowerScore Staff
  • Posts: 5153
  • Joined: Apr 14, 2011
|
#49730
Good analysis, Tomars! Although even if the stem had asked which one most weakens, that would still be a relative claim, and answer E would still be the best choice. Weakening the most just means the most as compared to the other answers, not compared to an outside standard of "the best possible answer of all possible answers." Not sure if that is what you were thinking when you raised the issue of a lower standard of proof vs airtight.

The author wants us to believe the high salaries are only attracting people driven by greed, to paraphrase the argument a bit. If most of the politicians are actually making less money than they otherwise would have in the private sector, that argument takes a pretty bad hit.

Nice work!
 andriana.caban
  • Posts: 142
  • Joined: Jun 23, 2017
|
#63549
Hi,

Why doesn't this argument not contain causal reasoning? I diagrammed the argument as:

High salaried officials (who, are attracted presumably only by the money) :arrow: Integrity decreased/effected
 Brook Miscoski
PowerScore Staff
  • PowerScore Staff
  • Posts: 418
  • Joined: Sep 13, 2018
|
#64275
Adriana,

I think you are trying to ask whether the argument contains causal reasoning. The first sentence explicitly asserts a cause-effect relationship, so the stimulus does contain causal reasoning. The remainder of the stimulus is devoted to arguing for that causal relationship.
 Katherinthesky
  • Posts: 36
  • Joined: Feb 07, 2020
|
#73734
I didn't choose E because even when current "Valitanian politicians could have obtained better-paid work outside outside politics," that doesn't mean those politicians aren't getting paid highly - i.e., they're STILL getting paid highly even if if they could be getting paid more outside of politics.

If the stimulus itself indicated that people who are primarily interested in making not only high incomes but the HIGHEST incomes associated with that of elected politicians in Valitania, then answer choice E would clearly weaken that argument.

But because the stimulus doesn't mention HIGHEST incomes but only HIGH incomes associated with Valitanian politicians, I can't see that E would necessarily weaken it since they're still getting high salaries regardless of whether or not they chose the even better-paid jobs outside, thus still satisfying the people who go in for the money and get into politics.
 James Finch
PowerScore Staff
  • PowerScore Staff
  • Posts: 943
  • Joined: Sep 06, 2017
|
#73739
Hi Katherine,

The stimulus allows us to make an inference that people interested primarily in money are "the wrong people" to be in politics. The stimulus then assumes that those wrong people are all Valitanian politicians, which leads to the conclusion about the disastrous effect the high salaries paid to politicians have had. So the easiest way to attack this argument is to make the assumption untrue, or at least likely to be untrue. This is what (E) does, by making it unlikely that most Valitanian politicians are primarily motivated by money. If you're primarily interested in money and could get a higher-paying job outside of politics, why would you be in politics?

Hope this clears things up!
 LSAT2020
  • Posts: 31
  • Joined: Jun 24, 2020
|
#76808
Tomars wrote:I debated between C and E. I understand why C is wrong now. Even though, the same number of folks compete when the salary is high or low, we do not necessarily have the same types of people. As the text says, it's still quite possible to have "the wrong [type] of people" who are more interested in making money.

But I'm still having problem justifying E because of the word "most". Even if "most" (e.g., 52%) current politicians could have been better paid elsewhere, there could still have been a significant enough number of folks (e.g., 48%) who joined because of the high pay and who could not as easily obtain better paying jobs elsewhere. In that case, it's still quite possible that the "wrong" type of people are drawn to being politicians in Valitania..those attracted primarily to the high salary.

And I imagine the burden of proof for this question type is not lower/has to be air tight, since it does not ask for which answer "most weakens".
I think the reason E still works here is because the stimulus seems to imply that all people in Valitania's country are attracted to politics because of the money. I think E creates doubt by saying that 51% or more of this group could've actually made more money, and thus 51% or more are not in it for the money. Even if the other 48% are in it for the money it still weakens the author's argument that they're ALL in it for the money. I'm still learning, so if anyone has any feedback for me I would greatly appreciate it :)
 Paul Marsh
PowerScore Staff
  • PowerScore Staff
  • Posts: 290
  • Joined: Oct 15, 2019
|
#80144
Hey LSAT2020! Thanks for giving your feedback to Tomars's question. Your response looks good to me. Just to add on to that - remember that for Weaken questions, we don't want to get too hung up on the "strength" of our answer choice. Our right answer doesn't need to 100% destroy every last shred of the argument, it just needs to make it less likely that the conclusion logically follows from the premises.

Nice one!
 Mastering_LSAT
  • Posts: 35
  • Joined: Jul 30, 2020
|
#92167
Hi there,

It’s clear to me why (A)-(D) are wrong, but I’m struggling to understand why (E) is correct, as it requires us to make several assumptions.

First, I agree with @Katherinthesky that “high” and “highest” salary are different concepts. The stimulus only talks about “…anyone whose primary aim in life is to make money…”, not to “…make more money…”, or “…as much money as possible…”. Therefore, why would such politicians necessarily go for another job that pays more? They have a high salary already, and not interested in getting the highest salary possible (at least the stimulus is silent about this possibility).

Are we required to make a basic assumption that those “…whose primary aim in life is to make money…” would absolutely go for a higher pay when the opportunity presents itself, even if the stimulus is silent about this possibility?


Second, the conclusion of the argument is talking about “elected politicians” but the correct answer choice about “politicians” only. If we are required to assume that “elected politicians” in the argument’s conclusion and “politicians” in the correct answer choice are the same concepts, I think it goes well beyond a basic assumption.

As an example, all politicians can be divided into elected and non-elected (e.g., appointed by elected politicians). Let us say that Valitania has 25 elected and 75 non-elected politicians. If most current Valitanian politicians could have obtained a better-paid jobs outside politics, it is possible that this answer choice could talk about the majority of non-elected politicians which would constitute the majority of all politicians in Valitania as well. However, we are not affecting our 25 elected politicians at all. The argument is talking about elected politicians, not all politicians. Therefore, depending on how we interpret “politicians” in the correct answer choice, this answer choice could be irrelevant and not weaken the argument even slightly.

Are we required to assume that “politicians” in the correct answer choice means “elected politicians” that the argument is talking about?

I would greatly appreciate it if you could clarify these two aspects. To Dave’s point in his blog, I’m not fighting with LSAT :-D I’m just trying to understand their mindset regarding the LSAT generally, and this question in particular.

Many thanks!
 Adam Tyson
PowerScore Staff
  • PowerScore Staff
  • Posts: 5153
  • Joined: Apr 14, 2011
|
#92188
Let's look to the question stem here, Mastering_LSAT, for a little help: "Which one of the following, if true, would weaken the argument?"

That question isn't asking us to disprove the conclusion. It's not even asking us which answer would do a lot to weaken the argument. It's just "which one weakens," which means the correct answer will weaken the argument and the wrong answers will not. For that reason, the bar is set very, very low for the correct answer. All it has to do is make us pause and think to ourselves "hmmm, that might be bad for the author." Even if we want to stop and gather more information, if the answer raises some doubt, that's enough!

The premises are not limited to elected politicians, but are about people who "have been attracted into Valitanian politics." You have a point that that may include appointed politicians in addition to elected ones, but we can still weaken the argument by raising doubts about the truth of the premise (not the most common way to attack an argument on this test, but still a valid approach). If you kick out one leg from under a table, the table is less stable - weaker - than it was before. Answer E does a pretty good job of sweeping that leg! If most of the politicians could have gotten a job elsewhere that pays more, that is a strong counter to the claim that those people are motivated primarily by a high salary and that they "are more interested in making money than in serving the needs of the nation." It sure looks like they are more service-oriented than the author gives them credit for! The author would absolutely be forced to defend themselves if that was brought up to counter their position. They couldn't just shrug it off and call it irrelevant!

Try looking at it this way: if the majority of current politicians, whether elected or appointed, are people who could have made more money doing something else, and who are therefore sacrificing at least some financial benefits in order to take these positions, what evidence is there to support the conclusion? Is there ANY evidence to back up that claim? I'm having a hard time seeing any. I could assume something else that might support it, but we should not be doing that kind of work to help the author out. A good argument stands on its own without our help, and this one, once we sweep that leg out from under it, doesn't appear to stand on anything at all.
 Mastering_LSAT
  • Posts: 35
  • Joined: Jul 30, 2020
|
#92193
Adam, terrific response! Thanks for your help.

Basically, the correct answer choice calls into question the author’s only support/premise ("...the wrong people must have been attracted into Valitanian politics: people who are more interested in making money than in serving the needs of the nation.") In other words, the correct answer choice says that if the current politicians were more interested in making money than serving the needs of the country, why wouldn’t they choose a better paying job instead? Consequently, the conclusion has a very shaky support now, if any support at all.

Get the most out of your LSAT Prep Plus subscription.

Analyze and track your performance with our Testing and Analytics Package.